Difference between revisions of "2001 AMC 12 Problems/Problem 3"

(Solution 2)
(Solution 1)
 
(20 intermediate revisions by 4 users not shown)
Line 1: Line 1:
 
{{duplicate|[[2001 AMC 12 Problems|2001 AMC 12 #3]] and [[2001 AMC 10 Problems|2001 AMC 10 #9]]}}
 
{{duplicate|[[2001 AMC 12 Problems|2001 AMC 12 #3]] and [[2001 AMC 10 Problems|2001 AMC 10 #9]]}}
  
== Problem ==
+
==Problem==
 
The state income tax where Kristin lives is levied at the rate of <math>p\%</math> of the first
 
The state income tax where Kristin lives is levied at the rate of <math>p\%</math> of the first
 
<math>\textdollar 28000</math> of annual income plus <math>(p + 2)\%</math> of any amount above <math>\textdollar 28000</math>. Kristin
 
<math>\textdollar 28000</math> of annual income plus <math>(p + 2)\%</math> of any amount above <math>\textdollar 28000</math>. Kristin
Line 7: Line 7:
 
annual income. What was her annual income?  
 
annual income. What was her annual income?  
  
<math>\text{(A)}\,\textdollar 28000 \qquad \text{(B)}\,\textdollar 32000 \qquad \text{(C)}\,\textdollar 35000 \qquad \text{(D)}\,\textdollar 42000 \qquad \text{(E)}\,\textdollar 56000</math>
+
<math>\textbf{(A)}\,\textdollar 28000 \qquad \textbf{(B)}\,\textdollar 32000 \qquad \textbf{(C)}\,\textdollar 35000 \qquad \textbf{(D)}\,\textdollar 42000 \qquad \textbf{(E)}\,\textdollar 56000</math>
  
== Solution==
+
==Solution 2==
  
=== Solution 1 ===
+
Let <math>A</math>, <math>T</math> be Kristin's annual income and the income tax total, respectively. Notice that
 
 
Let the income amount be denoted by <math>A</math>.
 
 
 
We know that <math>\frac{A(p+.25)}{100}=\frac{28000p}{100}+\frac{(p+2)(A-28000)}{100}</math>.
 
 
 
We can now try to solve for <math>A</math>:
 
 
 
<math>(p+.25)A=28000p+Ap+2A-28000p-56000</math>
 
 
 
<math>.25A=2A-56000</math>
 
 
 
<math>A=32000</math>
 
 
 
So the answer is <math>\boxed{B}</math>
 
 
 
=== Solution 2 ===
 
 
 
Let <math>A</math>, <math>T</math> be Kristin's annual income. Notice that  
 
 
<cmath>\begin{align*}
 
<cmath>\begin{align*}
p\%\cdot28000 + (p + 2)\%\cdot(A - 28000)</cmath> <cmath>= [p\%\cdot28000 + p\%\cdot(A - 28000)] + 2\%\cdot(A - 28000)</cmath> <cmath>= p\%\cdot A + 2\%\cdot(A - 28000)</cmath>
+
T &= p\%\cdot28000 + (p + 2)\%\cdot(A - 28000) \\
 +
&= [p\%\cdot28000 + p\%\cdot(A - 28000)] + 2\%\cdot(A - 28000) \\
 +
&= p\%\cdot A + 2\%\cdot(A - 28000)
 +
\end{align*}</cmath>
 +
We are also given that <cmath>T = (p + 0.25)\%\cdot A = p\%\cdot A + 0.25\%\cdot A</cmath>
 +
Thus, <cmath>p\%\cdot A + 2\%\cdot(A - 28000) = p\%\cdot A + 0.25\%\cdot A</cmath> <cmath>2\%\cdot(A - 28000) = 0.25\%\cdot A</cmath>
 +
Solve for <math>A</math> to obtain <math>A = \boxed{\textbf{(B) }\textdollar 32000} </math>.
  
\begin{align*}
+
~ Nafer
((2x+3)^3)' &= 3(2x+3)^2 \cdot (2x+3)' \\
 
&= 3(2x+3)^2 \cdot 2 \\
 
&= 6(2x+3)^2.
 
\end{align*}
 
  
 
== See Also ==
 
== See Also ==

Latest revision as of 01:25, 1 March 2024

The following problem is from both the 2001 AMC 12 #3 and 2001 AMC 10 #9, so both problems redirect to this page.

Problem

The state income tax where Kristin lives is levied at the rate of $p\%$ of the first $\textdollar 28000$ of annual income plus $(p + 2)\%$ of any amount above $\textdollar 28000$. Kristin noticed that the state income tax she paid amounted to $(p + 0.25)\%$ of her annual income. What was her annual income?

$\textbf{(A)}\,\textdollar 28000 \qquad \textbf{(B)}\,\textdollar 32000 \qquad \textbf{(C)}\,\textdollar 35000 \qquad \textbf{(D)}\,\textdollar 42000 \qquad \textbf{(E)}\,\textdollar 56000$

Solution 2

Let $A$, $T$ be Kristin's annual income and the income tax total, respectively. Notice that \begin{align*} T &= p\%\cdot28000 + (p + 2)\%\cdot(A - 28000) \\ &= [p\%\cdot28000 + p\%\cdot(A - 28000)] + 2\%\cdot(A - 28000) \\ &= p\%\cdot A + 2\%\cdot(A - 28000) \end{align*} We are also given that \[T = (p + 0.25)\%\cdot A = p\%\cdot A + 0.25\%\cdot A\] Thus, \[p\%\cdot A + 2\%\cdot(A - 28000) = p\%\cdot A + 0.25\%\cdot A\] \[2\%\cdot(A - 28000) = 0.25\%\cdot A\] Solve for $A$ to obtain $A = \boxed{\textbf{(B) }\textdollar 32000}$.

~ Nafer

See Also

2001 AMC 12 (ProblemsAnswer KeyResources)
Preceded by
Problem 2
Followed by
Problem 4
1 2 3 4 5 6 7 8 9 10 11 12 13 14 15 16 17 18 19 20 21 22 23 24 25
All AMC 12 Problems and Solutions
2001 AMC 10 (ProblemsAnswer KeyResources)
Preceded by
Problem 8
Followed by
Problem 10
1 2 3 4 5 6 7 8 9 10 11 12 13 14 15 16 17 18 19 20 21 22 23 24 25
All AMC 10 Problems and Solutions

The problems on this page are copyrighted by the Mathematical Association of America's American Mathematics Competitions. AMC logo.png